Difference between revisions of "2003 AMC 12A Problems/Problem 8"
Sevenoptimus (talk | contribs) (Combined Solutions 3 and 4, which are exactly the same, and improved notation, explanations, and grammar) |
|||
(5 intermediate revisions by 5 users not shown) | |||
Line 8: | Line 8: | ||
=== Solution 1=== | === Solution 1=== | ||
− | For | + | For any positive integer <math>n</math> which is not a perfect square, exactly half of its positive factors will be less than <math>\sqrt{n}</math>, since each such factor can be paired with one that is larger than <math>\sqrt{n}</math>. (By contrast, if <math>n</math> is a perfect square, one of its factors will be exactly <math>\sqrt{n}</math>, which would therefore have to be paired with itself.) |
− | Since <math>60</math> is not a perfect square, half of the positive factors of <math>60</math> | + | Since <math>60</math> is indeed not a perfect square, it follows that half of its positive factors are less than <math>\sqrt{60} \approx 7.746</math>. This estimate clearly shows that there are not even any integers, let alone factors of <math>60</math>, between <math>7</math> and <math>\sqrt{60}</math>. Accordingly, exactly half of the positive factors of <math>60</math> are in fact less than <math>7</math>, so the answer is precisely <math>\boxed{\mathrm{(E)}\ \frac{1}{2}}</math>. |
− | + | === Solution 2=== | |
+ | Testing all positive integers less than <math>7</math>, we find that <math>1</math>, <math>2</math>, <math>3</math>, <math>4</math>, <math>5</math>, and <math>6</math> all divide <math>60</math>. The prime factorization of <math>60</math> is <math>2^2 \cdot 3 \cdot 5</math>, so using the standard [[Divisor function|formula for the number of divisors]], the total number of divisors of <math>60</math> is <math>3 \cdot 2 \cdot 2 = 12</math>. Therefore, the required probability is <math>\frac{6}{12} = \boxed{\mathrm{(E)}\ \frac{1}{2}}</math>. | ||
− | + | ===Solution 3 (brute force)=== | |
+ | Though this is not recommended for reasons of time, one can simply write out all the factors of <math>60</math>, eventually finding that <cmath>60 = 1 \cdot 60 = 2 \cdot 30 = 3 \cdot 20 = 4 \cdot 15 = 5 \cdot 12 = 6 \cdot 10.</cmath> Hence <math>60</math> has <math>12</math> factors, of which <math>6</math> are less than <math>7</math> (namely, <math>1</math>, <math>2</math>, <math>3</math>, <math>4</math>, <math>5</math>, and <math>6</math>), so the answer is <math>\frac{6}{12} = \boxed{\mathrm{(E)}\ \frac{1}{2}}</math>. | ||
− | + | ==Video Solution== | |
− | + | https://youtu.be/C3prgokOdHc ~savannahsolver | |
− | + | ||
+ | https://www.youtube.com/watch?v=jpMzPl7vkxE ~David | ||
== See Also == | == See Also == | ||
Line 26: | Line 29: | ||
[[Category:Introductory Number Theory Problems]] | [[Category:Introductory Number Theory Problems]] | ||
+ | {{MAA Notice}} |
Latest revision as of 13:06, 20 June 2025
- The following problem is from both the 2003 AMC 12A #8 and 2003 AMC 10A #8, so both problems redirect to this page.
Contents
Problem
What is the probability that a randomly drawn positive factor of is less than
?
Solution
Solution 1
For any positive integer which is not a perfect square, exactly half of its positive factors will be less than
, since each such factor can be paired with one that is larger than
. (By contrast, if
is a perfect square, one of its factors will be exactly
, which would therefore have to be paired with itself.)
Since is indeed not a perfect square, it follows that half of its positive factors are less than
. This estimate clearly shows that there are not even any integers, let alone factors of
, between
and
. Accordingly, exactly half of the positive factors of
are in fact less than
, so the answer is precisely
.
Solution 2
Testing all positive integers less than , we find that
,
,
,
,
, and
all divide
. The prime factorization of
is
, so using the standard formula for the number of divisors, the total number of divisors of
is
. Therefore, the required probability is
.
Solution 3 (brute force)
Though this is not recommended for reasons of time, one can simply write out all the factors of , eventually finding that
Hence
has
factors, of which
are less than
(namely,
,
,
,
,
, and
), so the answer is
.
Video Solution
https://youtu.be/C3prgokOdHc ~savannahsolver
https://www.youtube.com/watch?v=jpMzPl7vkxE ~David
See Also
2003 AMC 10A (Problems • Answer Key • Resources) | ||
Preceded by Problem 7 |
Followed by Problem 9 | |
1 • 2 • 3 • 4 • 5 • 6 • 7 • 8 • 9 • 10 • 11 • 12 • 13 • 14 • 15 • 16 • 17 • 18 • 19 • 20 • 21 • 22 • 23 • 24 • 25 | ||
All AMC 10 Problems and Solutions |
2003 AMC 12A (Problems • Answer Key • Resources) | |
Preceded by Problem 7 |
Followed by Problem 9 |
1 • 2 • 3 • 4 • 5 • 6 • 7 • 8 • 9 • 10 • 11 • 12 • 13 • 14 • 15 • 16 • 17 • 18 • 19 • 20 • 21 • 22 • 23 • 24 • 25 | |
All AMC 12 Problems and Solutions |
These problems are copyrighted © by the Mathematical Association of America, as part of the American Mathematics Competitions.